$\sqrt{\|.\|_1\|.\|_2}$ est une norme ? — Les-mathematiques.net The most powerful custom community solution in the world

$\sqrt{\|.\|_1\|.\|_2}$ est une norme ?

Bonjour

J'ai essayé de montrer que $N = \sqrt{ \| \cdot \| _1 \| \cdot \| _2 } : x \mapsto \sqrt{ \| x \| _1 \| x \| _2 }$ était une norme sur $\mathbb R^2$ après avoir essayé de nier l'inégalité triangulaire pour diverses valeurs, hélas sans succès. L'inégalité triangulaire induite m'a alors l'air un peu difficile, aussi, j'en appelle à votre aide !

L'idée initiale était de montrer qu'il existait des normes $N_1$ et $N_2$ non proportionnelles telles que $\sqrt{ N_1 N_2} : x \mapsto \sqrt{ N_1(x) N_2(x)} $ était une norme.

Pourriez-vous me dire si $N$ est une norme ou non ? Si non, existe-t-il des normes $N_1$ et $N_2$ vérifiant les hypothèses ci-dessus ?
Merci d'avance !

Réponses

  • Bonjour
    Je ne pense pas que N vérifie l'inégalité triangulaire.
    exemple avec $x= (1,1)$ et $y=(1,0)$
     
  • Justement, j'avais d'abord essayé avec ces deux vecteurs mais je trouve bien $N(x+y) = \sqrt{3\sqrt 5} \leq \sqrt{2\sqrt 2} + 1= N(x) + N(y)$, sauf erreur de ma part.
  • Je me suis trompé de sens. Alors il n'y a pas de contre exemple évident
    et il se peut bien que N soit une norme.
     
  • Je n'ai pas fait d'exo de maths depuis fort longtemps et j'essaye de m'y remettre en lisant ce forum donc désolé si je dis des bêtises.
    Je n'ai pas la solution mais une intuition pour prouver que ce n'est pas une norme voici mon idée si quelqu'un arrive à finir :
    Déjà pour simplifier les racines je propose de comparer $(N(x) + N(y))^2$ et $N(x+y)^2$
    Je trouve $(N(x) + N(y))^2 = ||x||_1 ||x||_2 + ||y||_1 ||y||_2 + 2\sqrt{||x||_1 ||x||_2||y||_1 ||y||_2}$
    et $N(x+y)^2 = ||x+y||_1||x+y||_2 \leq (||x||_1 + ||y||_1)(|x||_2 + ||y||_2) = ||x||_1 ||x||_2 + ||y||_1 ||y||_2 + ||x||_1||y||_2 + ||y||_1||x||_2$

    Si on pose $a=||x||_1||y||_2$ et $b=||y||_1||x||_2$ les derniers termes de mes lignes précédentes sont $2*\sqrt{ab}$ et $a+b$
    Or on sait que la moyenne arithmétique est supérieure ou égale à la moyenne géométrique ($\sqrt{ab}\leq(\frac{a+b}{2}$) donc si on arrive se placer dans un cas d'égalité de la majoration que j'ai faite (inégalité triangulaire sur les normes 1 et 2 en même temps) on aurait la moyenne arithmétique du côté de $N(x+y)$ et la moyenne géométrique de l'autre et donc potentiellement $N(x+y) > N(x) + N(y)$.
    Mais c'est peut-être une fausse piste ...
  • Bonsoir,

    J'ai l'impression que vous majorez $N(x+y)^2$ alors qu'en fait vous vouliez minorer cette quantité...
  • Ok l'idée était un peu tordu mais oui c'est bien ce que j'ai fait. En fait je faisais une majoration en remarquant que dans le cas où ma majoration est une égalité on se retrouvait avec une expression qui contient une moyenne arithmétique à comparer à l'autre expression avec une moyenne géométrique et du coup on aurait pu trouver à partir de là un vecteur qui contredise l'inégalité triangulaire pour N. Mais je viens de me rafraîchir la mémoire sur le cas d'égalité de l'inégalité triangulaire que j'ai utilisée, il faut que les vecteurs x et y soit proportionnels mais du coup ça ne conduit pas au contre-exemple que j'espérais...
  • Bonjour, il suffit de considérer les deux sphères unités : pour $\|. \|_1$ c'est le carré principal de côté de longueur $1$, centré en $0$, auquel on a appliqué une rotation d'angle $\pi/4$ radian inscrit dans la sphère unité pour $\| . \|_2$ : un cercle contré à l'origine de rayon $1$. Sur la sphère pour $\|.\|_1$ on a $\vert\vert x\vert\vert_2\ge \sqrt2/2$. On déduit de $\forall x,~x=\vert\vert x\vert\vert_1 (x/\vert\vert x\vert\vert_1)$ que $\forall x,~\vert\vert x\vert\vert_1\ge (\sqrt2/2) \vert\vert x\vert\vert_2$.
    Ensuite considérer que la sphère pour $\|.\|_2$ est inscrite dans le carré principal de côté $\sqrt2$ auquel on a appliqué une rotation d'angle $\pi/4$ radians et en déduire que $\forall x,~\vert\vert x\vert\vert_2\leq\sqrt2\vert\vert x\vert\vert_1$ pour obtenir au final $\forall x,~(\sqrt2/2)\vert\vert x\vert\vert_2\leq\vert\vert x\vert\vert_1\leq\sqrt2\vert\vert x\vert\vert_1$.
    L'équivalence des normes étant obtenue développer $N(x+y)^2$...
    Les mathématiques ne sont pas vraies, elles sont commodes.
    Henri Poincaré
  • Et donc ça marcherait pour toutes les normes ?...
  • Quelque-soit la norme, je ne sais pas et je ne pense pas. D'ailleurs si dans la démonstration on fait intervenir l'inégalité triangulaire des normes 1 et 2 (et plus généralement de 2 normes $n_1$ et $n_2$) on n'obtient pas l'inégalité.

    Evidemment j'ai essayé l'équivalence des 2 normes en majorant- minorant et en faisant intervenir les constantes $c_1$ et $c_2,$ et ceci de différentes façons. Mais sauf erreur à chaque fois cela n'aboutit pas.

    C'est à dire que je veux bien croire en l'idée de @AlainLyon mais il faudrait qu'il montre les détails.

    J'en suis arrivé à penser que dans ce cas particulier N est une norme mais pour le démontrer il faut écrire l'inégalité

    $N(x+y)^2\leq (N(x)+N(y))^2$ (1)

    la prouver (sans faire intervenir $n_i(x+y)\leq n_i(x)+n_i(y)$ sinon c'est bloqué)

    C'est à dire (1) est une inégalité faisant intervenir 4 réels (les coordonnées de x et y) ,
    inégalité un peu tordue ....
     
  • Bonjour,

    Merci pour votre intérêt porté à ma question.

    On aimerait procéder ainsi dans le cas quelconque, où $N : x \mapsto \sqrt{N_1(x) N_2(x)}$.
    $N(x+y) = \sqrt{N_1(x+y) N_2(x+y)} \leq \sqrt{(N_1(x) + N_1(y)) (N_2(x) + N_2(y))}$. Le membre de droite est alors inférieur à $N(x) + N(y)$ si et seulement si $ac+ad+bc+bd \leq ac + bd + 2 \sqrt{abcd} \iff ad = bc$, où $a=N_1(x), b= N_1(y), c = N_2(x), d= N_2(y)$.
    Ainsi, $\sqrt{(N_1(x) + N_1(y)) (N_2(x) + N_2(y))} \leq N(x) + N(y)$ pour tout $(x,y)$ si et seulement si $N_1$ et $N_2$ sont proportionnelles.

    J'ai donc naturellement envie de penser que pour certaines normes $N_1$ et $N_2$ non proportionnelles, $N$ est tout de même une norme (à cause de la première majoration). Il est toutefois relativement facile de trouver des normes $N_1$ et $N_2$ pour lesquelles l'application $N$ associée n'est pas une norme (comme $N_1 = \| \cdot \| _1$ et $N_2 = \| \cdot \| _{\infty }$ ou encore $N_1 : (x,y) \mapsto |x| + 2 |y|$ et $N_2 : (x,y) \mapsto N_1(y,x)$, sauf erreur de calcul de ma part...). Toutefois, je n'ai pas trouvé de contre-exemple simple pour le cas $N_1 = \| \cdot \| _1$ et $N_2 = \| \cdot \| _{2}$.

    @AlainLyon n'a-t-on pas plutôt $\| x \| _2 \leq \| x \| _1 \leq \sqrt{2} \| x \| _2$ ? Quel est l'intérêt de cet encadrement ? A trouver un contre-exemple ou à montrer qu'il s'agit d'une norme ?

    @Eusebius Je vois.
  • L'inégalité (1) est équivalente à $(a+b+c+d) \sqrt{a^2 + b^2 + c^2 + d^2 + 2(ac+bd)} \leq (a+b)\sqrt{a^2 + b^2} + (c+d) \sqrt{c^2 + d^2} + 2\sqrt{(a+b)(c+d)\sqrt{(a^2 + b^2)(c^2 + d^2)}}$, en posant $x=(a,b)$ et $y=(c,d)$.

    Muirhead ou Schur s'y cachent peut-être...
  • Non c'est plus compliqué que ça. a,b,c,d peuvent avoir des signes négatifs...
     
  • $a,b,c,d$ désignent en fait les valeurs absolues des coordonnées respectives de $x$ et $y$. On peut donc se restreindre à $(x,y) \in \left( \mathbb R_+\right) ^2$, non ?

    EDIT : Donc, il faudrait plutôt montrer $(a+b+c+d) \sqrt{a^2 + b^2 + c^2 + d^2 + 2(\pm ac+ \pm bd)} \leq (a+b)\sqrt{a^2 + b^2} + (c+d) \sqrt{c^2 + d^2} + 2\sqrt{(a+b)(c+d)\sqrt{(a^2 + b^2)(c^2 + d^2)}}$, en posant $x=(a,b)$ et $y=(c,d)$, avec $a,b,c,d \geq 0$, toutefois le membre de gauche est maximal quand $\pm ac$ et $\pm bd$ sont positifs, donc il suffit de montrer que (1) est vraie sans les signes $\pm $ ?
  • Non on ne peut pas se restreindre à $(\R^+)^2$ Je crois qu'il faut envisager touts les cas.
    À moins que quelqu'un n'apporte une solution nouvelle, je ne vois que cette possibilité et cela devient assez long, voire compliqué !

    L'origine du sujet c'est quoi ?
     
  • Peut-être as-tu raté mes éditions, j'y explique pourquoi je pense que l'on peut se restreindre à $a,b,c,d \geq 0$. En considérant en fait les valeurs absolues des coordonnées $a',b',c',d'$, on constate que la première inégalité que j'ai écrite doit être vérifiée par $a',b',c',d'$ avec deux signes $\pm$, c'est-à-dire : $(a'+b'+c'+d') \sqrt{a'^2 + b'^2 + c'^2 + d'^2 + 2(\pm a'c'+ \pm b'd')} \\ \hspace{1cm} \leq (a'+b')\sqrt{a'^2 + b'^2} + (c'+d') \sqrt{c'^2 + d'^2} + 2\sqrt{(a'+b')(c'+d')\sqrt{(a'^2 + b'^2)(c'^2 + d'^2)}}$,
    en posant donc $a' = |a|, b' = |b|, c' = |c|,$ et $d' = |d|$.
    Toutefois, le membre de gauche est maximal lorsque les symboles $\pm$ sont des signes +, donc il suffit de montrer cette inégalité sans les symboles $\pm$.

    Il n'y a pas d'origine, désolé... je me demandais juste si le produit de deux normes était également une norme en rédigeant un mini-cours sur les normes pour un ami.
  • Le probleme est donc de montrer que l'ensemble $C=\{(x,y); (|x|+|y|)^2(x^2+y^2)<1\}$ est convexe. Il est limite par la courbe d'equation en coordonnees polaires $$\rho(\theta)=(|\cos\theta|+|\sin\theta|)^{-1/4}.$$La concavite est tournee vers le pole en $\theta$ si et seulement si
    $$c(\theta)=\frac{1}{\rho(\theta)}+\left(\frac{1}{\rho(\theta)}\right)''>0$$ (voir Lelong-Ferrand et Arnaudies tome 3). Ici pour $\theta\in]0,\pi/2[$ on a
    $$c(\theta)=(\cos\theta+\sin\theta)^{-3/4})\left(\frac{19}{16}\cos \theta+\frac{13}{16}\sin \theta\right)>0.$$
  • @P. (tu)
    La boule convexe en image.

    [Contenu du pdf joint. AD]127068
     
  • Merci beaucoup :-)

    J'ai juste une petite interrogation : est-ce une puissance $\frac{-1}{2}$ plutôt que $\frac{-1}{4}$ ? J'ai peut-être fait une erreur dans mon calcul.
  • Oui. L'équation de la courbe c'est N(x)=1 donc tu mets une puissance -1/2. Le calcul est différent mais le résultat
    devrait être identique. Attention tout de même a bien argumenter la preuve car l'équation contient des valeurs
    absolues et ainsi la courbe n'est pas de classe $C^2$ en certains points. C'est-à-dire que le rayon de courbure n'est que continue par morceaux sur $[0,2\pi].$
     
  • Effectivement, le calcul est même un peu plus facile.
    On voit sur ton dessin que la courbe n'est même pas dérivable, mais cela n'a pas d'importance.
    Je soutiens d'ailleurs toujours que l'on peut se restreindre au quadrant supérieur droit. En plus, on voit très bien les deux axes de symétrie $x=0$ et $y=0$ sur ton dessin !
  • @Seb Baumert
    Vu toutes les symétries, on peut en effet se restreindre à l'étude de la concavité de la courbe dans le quadrant supérieur droit, et même à l'étude en polaire sur [0;pi/4], mais à condition d'y rajouter la vérification de la valeur de la pente de la courbe au point (1,0), qui doit être négative sans quoi la convexité de l'intérieur n'est plus vraie...
    Bon vérification faite, ça marche bien ici : sauf erreur de calcul, la pente en (1,0) vaut -1/2.
    .
  • OK pour le 1/2 a la place du 1/4, merci.
  • Je suis d'accord. Je trouve bien $\rho '(0) = - \frac{1}{2}$ donc la pente (en différenciant y par rapport à x) sur le dessin en $(1,0)$ devrait valoir -2, ce qui a l'air cohérent.

    Merci à tous ! :-)
Connectez-vous ou Inscrivez-vous pour répondre.
Success message!